You are on page 1of 51

Physics 831: Statistical Mechanics

Russell Bloomer
1
University of Virginia
Note: There is no guarantee that these are correct, and they should not be copied
1
email: rbloomer@virginia.edu
Contents
1 Problem Set 1 1
1.1 Kittel 8.1: Heat pump . . . . . . . . . . . . . . . . . . . . . . . . . . . . . . . . . . . . . . . . 1
1.2 Kittel 8.6: Room air conditioner . . . . . . . . . . . . . . . . . . . . . . . . . . . . . . . . . . 1
1.3 Kittel 8.9: Cooling of nonmetallic solid to T = 0 . . . . . . . . . . . . . . . . . . . . . . . . . 2
1.4 Sterling Heat Engine . . . . . . . . . . . . . . . . . . . . . . . . . . . . . . . . . . . . . . . . . 2
1.5 Unavailability for work . . . . . . . . . . . . . . . . . . . . . . . . . . . . . . . . . . . . . . . . 3
1.6 Gibbs Free Energy . . . . . . . . . . . . . . . . . . . . . . . . . . . . . . . . . . . . . . . . . . 3
2 Problem Set 2 5
2.1 Spin model . . . . . . . . . . . . . . . . . . . . . . . . . . . . . . . . . . . . . . . . . . . . . . 5
2.2 Paramagnetism of a system of N localized spin-1/2 particles . . . . . . . . . . . . . . . . . . . 6
2.3 Kittel 2.3: Quantum harmonic oscillator . . . . . . . . . . . . . . . . . . . . . . . . . . . . . . 7
2.4 Review Thermal mechanics . . . . . . . . . . . . . . . . . . . . . . . . . . . . . . . . . . . . . 7
2.5 Review Thermal mechanics . . . . . . . . . . . . . . . . . . . . . . . . . . . . . . . . . . . . . 8
3 Problem Set 3 9
3.1 Kittel 3.2: Magnetic susceptibility . . . . . . . . . . . . . . . . . . . . . . . . . . . . . . . . . 9
3.2 Kittel 3.3: Free energy of a harmonic oscillator . . . . . . . . . . . . . . . . . . . . . . . . . . 10
3.3 Kittel 3.4: Energy uctuations . . . . . . . . . . . . . . . . . . . . . . . . . . . . . . . . . . . 10
3.4 Kittel 3.10: Elasticity of polymers . . . . . . . . . . . . . . . . . . . . . . . . . . . . . . . . . 11
3.5 Ising spin chain . . . . . . . . . . . . . . . . . . . . . . . . . . . . . . . . . . . . . . . . . . . . 12
4 Problem Set 4 13
4.1 Application of equal partition theorem . . . . . . . . . . . . . . . . . . . . . . . . . . . . . . . 13
4.2 Thermal Equilibrium of the Sun and Earth . . . . . . . . . . . . . . . . . . . . . . . . . . . . 14
4.3 Kittel 4.3: Average temperature of the interior of the Sun . . . . . . . . . . . . . . . . . . . . 14
4.4 Kittel 4.6: Pressure of thermal radiation . . . . . . . . . . . . . . . . . . . . . . . . . . . . . . 15
4.5 Kittel 4.7: Free energy of a photon gas . . . . . . . . . . . . . . . . . . . . . . . . . . . . . . . 15
4.6 Kittel 4.18: Isentropic expansion of photon gas . . . . . . . . . . . . . . . . . . . . . . . . . . 16
5 Problem Set 5 17
5.1 5.1: Kittel 4.14: Heat capacity of liquid
4
He at low temperature . . . . . . . . . . . . . . . . . . . 17
5.2 5.2: Kittel 4.15: Angular distribution of radiant energy ux . . . . . . . . . . . . . . . . . . . . . . 17
5.3 5.3: Qualifying exam le problem . . . . . . . . . . . . . . . . . . . . . . . . . . . . . . . . . . . 17
5.4 5.4: Helmholtz free energy for the Debye model . . . . . . . . . . . . . . . . . . . . . . . . . . . . 17
5.5 5.5: Kittel 5.3: Potential energy of gas in gravitational eld . . . . . . . . . . . . . . . . . . . . . . 17
i
6 Problem Set 6 19
6.1 6.1: Kittel 5.4: Active transport . . . . . . . . . . . . . . . . . . . . . . . . . . . . . . . . . . . . 19
6.2 6.2: Kittel 5.7: States of positive and negative ionization . . . . . . . . . . . . . . . . . . . . . . . 19
6.3 6.3: Kittel 5.10: Concentration uctuation . . . . . . . . . . . . . . . . . . . . . . . . . . . . . . 20
6.4 6.4: Kittel 6.3: Distribution function for double occupancy statistics . . . . . . . . . . . . . . . . . 20
6.5 6.5: Kittel 6.7: Relation of pressure and energy density . . . . . . . . . . . . . . . . . . . . . . . . 21
6.6 6.6: Kittel 6.9: Gas of atoms with internal degree of freedom . . . . . . . . . . . . . . . . . . . . . 21
7 Problem Set 7 23
7.1 7.1: Pressure in types of gases . . . . . . . . . . . . . . . . . . . . . . . . . . . . . . . . . . . . . 23
7.2 7.2: Kittel 5.13: Isentropic expansion . . . . . . . . . . . . . . . . . . . . . . . . . . . . . . . . . 23
7.3 7.3: Kittel 6.8: Time for a large uctuation . . . . . . . . . . . . . . . . . . . . . . . . . . . . . . 23
7.4 7.4: Kittel 6.10: Isentropic relations of ideal gas . . . . . . . . . . . . . . . . . . . . . . . . . . . . 23
7.5 7.5: Kittel 6.12: Ideal gas in two dimensions . . . . . . . . . . . . . . . . . . . . . . . . . . . . . . 23
7.6 7.6: Kittel 7.4: Chemical potential versus temperature . . . . . . . . . . . . . . . . . . . . . . . . 23
7.7 7.7: The absorbtion of gas onto a surface . . . . . . . . . . . . . . . . . . . . . . . . . . . . . . . 23
8 Problem Set 8 25
8.1 8.1: Mixing of two distinct atoms . . . . . . . . . . . . . . . . . . . . . . . . . . . . . . . . . . . 25
8.2 8.2: Kittel 7.3: Pressure and entropy of degenerate Fermi gas . . . . . . . . . . . . . . . . . . . . . 25
8.3 8.3: Kittel 7.6: Mass-radius relationship for white dwarfs . . . . . . . . . . . . . . . . . . . . . . . 26
8.4 8.4: Kittel 7.10: Relativistic white dwarfs stars . . . . . . . . . . . . . . . . . . . . . . . . . . . . 27
8.5 8.5: Electrons in the air o a conductor . . . . . . . . . . . . . . . . . . . . . . . . . . . . . . . . 27
9 Special Problem Set (9) 29
9.1 Problem 1: Properties of Photon Gas . . . . . . . . . . . . . . . . . . . . . . . . . . . . . . . . 29
9.2 Problem 2: Engine Cycle . . . . . . . . . . . . . . . . . . . . . . . . . . . . . . . . . . . . . . . . 29
9.3 Problem 3: Vibrational Modes of a Molecule . . . . . . . . . . . . . . . . . . . . . . . . . . . . . 30
9.4 Problem 4: Relativistic Massless Bosons . . . . . . . . . . . . . . . . . . . . . . . . . . . . . . . . 31
9.5 Problem 5: Kittel & Kroemer 7.9 and more . . . . . . . . . . . . . . . . . . . . . . . . . . . . . . 32
10 Problem Set 10 33
10.1 10.1: Collisions with a wall for a Fermi Gas . . . . . . . . . . . . . . . . . . . . . . . . . . . . . . 33
10.2 10.2: Free energy and pressure of a Boson gas . . . . . . . . . . . . . . . . . . . . . . . . . . . . . 33
10.3 10.3: Discontinuity in the slope of the heat capacity of a Bose gas . . . . . . . . . . . . . . . . . . 34
10.4 10.4: Maximum work extracted from an ideal gas . . . . . . . . . . . . . . . . . . . . . . . . . . . 34
11 Problem Set 11 37
11.1 11.1:A review problem . . . . . . . . . . . . . . . . . . . . . . . . . . . . . . . . . . . . . . . . . 37
11.2 11.2: Dissociation of water . . . . . . . . . . . . . . . . . . . . . . . . . . . . . . . . . . . . . . . 37
11.3 11.3: Practice with the Jacobian . . . . . . . . . . . . . . . . . . . . . . . . . . . . . . . . . . . . 38
11.4 11.4: More practice . . . . . . . . . . . . . . . . . . . . . . . . . . . . . . . . . . . . . . . . . . 39
11.5 11.5: van der Waals Gas . . . . . . . . . . . . . . . . . . . . . . . . . . . . . . . . . . . . . . . . 39
12 Problem Set 12 41
12.1 12.1: Maxwell Relations . . . . . . . . . . . . . . . . . . . . . . . . . . . . . . . . . . . . . . . . 41
12.2 12.2: Equilibrium conditions . . . . . . . . . . . . . . . . . . . . . . . . . . . . . . . . . . . . . . 41
12.3 12.3: Fluctuation in number of a Fermi gas . . . . . . . . . . . . . . . . . . . . . . . . . . . . . . 42
12.4 12.4: Fluctuation in volume, pressure, entropy and temperature . . . . . . . . . . . . . . . . . . . 42
12.5 12.5: Kittel 10.5: Gas-solid equilibrium . . . . . . . . . . . . . . . . . . . . . . . . . . . . . . . . 42
ii
13 Problem Set 13 45
13.1 13.1: Superconduction and Heat Capacity . . . . . . . . . . . . . . . . . . . . . . . . . . . . . . . 45
13.2 13.2: Kittel 10.8: First order crystal transformation . . . . . . . . . . . . . . . . . . . . . . . . . . 45
13.3 13.3: Kittel 11.2: Mixing energy in
3
He
4
He and Pb Sn mixtures . . . . . . . . . . . . . . . . 46
13.4 13.4: Kittel 11.4: Solidication range of a binary alloy . . . . . . . . . . . . . . . . . . . . . . . . 46
13.5 13.5: Kittel 11.5: Alloying of gold into silicon . . . . . . . . . . . . . . . . . . . . . . . . . . . . . 47
iii
Chapter 1
Problem Set 1
1.1 Kittel 8.1: Heat pump
(a) Show that for a reversible heat pump the energy required per unit of heat delivered inside the building is given
by the Carnot eciency
W
Q
h
= C =

h

h
(1.1)
What happens if the heat pump is not reversible?
For a reversible system:
h
=
l
. From the denition of entropy
l
=
Q
l

l
;
h
=
Q
h

h
. Then
W
Q
h
=
Q
h
Q
l
Q
h
=

h

h
=
l

h
= C
(b) Assume that the electricity consumed by a reversible heat pump must itself be generated by a Carnot engine
operating between the temperatures
hh
and
l
. What is the ratio Q
hh
/Q
h
, of the beat consumed at
hh
, to
the heat delivered at
h
? Give numerical values for T
hh
= 600 K; T
h
= 300 K; T
l
= 270 K.
For a Carnot engine
W
Q
hh
=

hh

hh
W =

hh

hh
Q
hh
The ratio is then
Q
hh
Q
h
=

hh
W/(
hh

l
)

h
W/(
h

l
)

Q
hh
Q
h
=
(
h

l
)/
h
(
hh

l
)/
hh

Q
hh
Q
h
=
1

l

h
1

l

hh
For
hh
= 600 K,
h
= 300 K and
l
= 270 K,
1
270
300
1
270
600
= .18
(c) Draw an energy-entropy ow diagram for the combination heat engine-heat pump, similar to Figures 8.1, 8.2
and 8.4, but involving no external work at all, only energy and entropy ows at three temperatures.
1.2 Kittel 8.6: Room air conditioner
A room air conditioner operates as a Carnot cycle refrigerator between an outside temperature T
h
and a room at a
lower temperature T
l
. The room gains heat from the outdoors at a rate A(T
h
T
l
); this heat is removed by the air
conditioner. The power supplied to the cooling unit is P.
(a) Show that the steady state temperature of the room is
1
T
l
= (T
h
+P/2A)
_
(T
h
+P/2A)
2
T
2
h

1/2
(1.2)
The rate is
dQ
l
dt
= A(T
h
T
l
), so the power is
P =
dW
dt
=
1

dQ
dt
= A
(T
h
T
l
)
2
T
l
Solving for T
l
T
2
l
(2T
h
P/A) T
l
+T
2
h
= 0 T
l
= T
h
+
P
2A

_
(T
h
+P/2A)
2
T
2
h

(b) If the outdoors is at 37
o
C and the room is maintained at 17
o
C by a cooling power 2 kW, nd the heat loss
coecient A of the room in WK
1
.
290 = 310 +
1000
A

_
310 +
1000
A
_
2
310
2
400
40000
A
+
(1000)
2
A
2
= 310
2
+
620000
A
+
(1000)
2
A
2
310
2
A = 1450 W/K
1.3 Kittel 8.9: Cooling of nonmetallic solid to T = 0
Wa saw in Chapter 4 that the heat capacity of nonmetallic solids at suciently low temperatures is proportional to
T
3
, as C = aT
3
. Assume it were possible to cool a piece of such a solid to T = 0 by means of a reversible refrigerator
that uses the solid specimen as its (varying!) low-temperature reservoir, and for which the high-temperature reservoir
has a xed temperature T
h
equal to the initial temperature Ti of the solid. Find an expression for the electrical energy
required.
Rate of change in work is dW = d(Q
h
Q
l
) = dQ
l
=
T
h
T
l
T
l
dQ
l
. We are given C = aT
3
dQ
l
= dT
l
dQ
l
= aT
3
l
dT
l
Therefore the work is
dW =
a
T
l
(T
h
T
l
) T
3
l
dT
l
For cooling T
h
0
_
W = a
_
0
T
h
_
T
h
T
2
l
T
3
l
_
dT
l
W =
a
3
T
h
T

0
T
h
+
a
4
T
4

0
T
h
W =
a
12
T
4
h

1.4 Sterling Heat Engine
The operation of a certain type of engine involves applying two isothermal steps and two isovolumetric steps per cycle
to one mole of diatomic gas. The largest and smallest volumes shown are VL and VS, respectively. You do not need
to consider the vibrational degree of freedom of the molecules.
(a) Find the eciency of the engine in terms of T
h
, Tc, VL and VS.
From 1 2 the process is isothermal so pV = constant, so the work is
Q12 =
_
pdV Q12 =
_
V
L
V
S
cdV
V
= c ln V

V
L
V
S
Q12 = nRT
h
ln
_
VL
VS
_
The heat along 2 3: Q23 =
5
2
R(Tc T
h
). For 3 4: Q34 = nRTc ln
_
V
L
V
S
_
. Finally, for 4 1: Q41 =
5
2
R(T
h
Tc). Then
W
Q
h
=
nRT
h
ln
_
V
L
V
S
_
+
5
2
R(T
h
Tc) +
5
2
R(Tc T
h
) nRTc ln
_
V
L
V
S
_
nRT
h
ln
_
V
L
V
S
_
+
5
2
R(T
h
TL)
=
T
h
ln
_
V
L
V
S
_
Tc ln
_
V
L
V
S
_
T
h
ln
_
V
L
V
S
_
+
5
2
(T
h
Tc)

2
(b) Prove that this engine has a lower eciency than that of Carnot engine operating at the highest and lowest
temperatures in the cycle.
=
(T
h
Tc) ln
_
V
L
V
S
_
5
2
(T
h
Tc) +T
h
ln
_
V
L
V
S
_ =
1
Tc
T
h
1 +
5/2
ln(V
L
/V
S
)
_
1
Tc
T
h
_ =
C
1 +
5
2

C
ln(V
L
/V
S
)
where C is the Carnots eciency. Now
5
2
C
ln (VL/VS)
> 0
because C > 0 and VL > VS ln (VL/VS) > 0. < C
1.5 Unavailability for work
A certain engine works in a cycle between reservoirs at T
h
and Tc. Find the dierence in the work dome by an
otherwise ideal engine (Carnot engine) and that done by the above engine in terms of SU, where SU is the change
in entropy in the universe due to the operation of the engine.
The change in the entropy on the T
h
is dU
h
= dQ
h
=
h
d
h
. The low side, dUc = dQc = cdc. By conservation
dQ
h
+dQc = 0. Then dirr = d
h
+dc = dQ
h
/
h
+dQc/c. Then
dirr =
_
1

1
c
_
dQ
h
=
c
h

h
c
dQ
h
The heat ow is independent of the temperature, so solving for Q
h
irr =
c
h

h
c
Q
h
Q
h
=

h
c
c
h
irr
An ideal engine has W = CQ
h
where C =

h
c

h
. Substituting the change in entropy to nd the change in work
W =

h
c

h
c
c
h
irr W = cirr
For the universe the sign is change, so in conventional units
W = TcSU
1.6 Gibbs Free Energy
The Gibbs free energy of a certain system is given by the follow formula:
G = n ln
_
a
5/2
P
_
(1.3)
where a is a constant. Compute
(a) the entropy
G = U +pV
_
G

_
pV
= = N ln
_
a
5/2
p
_
+N
5
2
1

= N ln
_
a
5/2
p
_
+
5
2
N
(b) the heat capacity at constant P, ie Cp
Cp =
_

_
p
=

_
= N ln
_
a
5/2
p
_
+
5
2
N
_
=
_
5N
2
_
Cp =
5
2
N
3
(c) the equation of state relating P, V , and
V =
_
G
p
_
= N
_
1
p
_
pV = N
(d) the energy
G = U +pV U = G+ pV = N ln
_
a
5/2
p
_
+
5
2
N N N ln
_
a
5/2
p
_
U =
3
2
N
(e) the chemical potential
=
_
G
N
_
p
= ln
_
a
5/2
p
_

4
Chapter 2
Problem Set 2
2.1 Spin model
In an isolated system of a large number N of localized particles of spin 1/2, each particle has a magnetic moment of
which can point either parallel or antiparallel to an applied magnetic eld H. The energy of the system when n
and n is given by U = (n n )H. Consider the energy range between U and U +U where U is very much
smaller than U but may be microscopically large so that U H.
(a) What is the total number of states (U) lying in this energy range?
This problem is much like the random walk problem in that there are only two options for the spins to align along
and that they are equally as likely. With that, the multiplicity is a binomial distribution. Dening m n n ,
the energy becomes U = mH. From the random walk
(m) =
N!
_
N+m
2
_
!
_
Nm
2
_
!
Solving m in terms of the energy m =
U
H
, the distribution becomes
(U) =
N!
_
N
U
H
2
_
!
_
N+
U
H
2
_
!
Because U is small compared to the number of available state and that is much greater than H, U is the width of
the distribution. This has to be unit of the number, so it becomes U
U
2H
. The nal solution for U +U becomes
(U) =
N!
_
N
U
H
2
_
!
_
N+
U
H
2
_
!
U
2H

(b) Assume that the energy U is in a region where (U) is appreciable. Apply the Gaussian approximation to part
(a) to obtain a simple expression for (U)dU as a function of U.
To nd the Gaussian approximation the logarithm of needs to be expanded. The expansion of a binomial is given
by Equation 2.34
1
.
log g

=
1
2
log 2/N +N log 2 2s
2
/N (2.1)
For this problem s =
m
2
=
U
2H
. This approximation becomes
log ((U) +U)

=
1
2
log 2/N +N log 2
2
_
U
2H
_
2
/N + log
U
2H
U goes to dU as the function becomes continuous. This should become a Gaussian once the removal of logarithm.
(U)dU = (2/N)
1/2
2
N
e
2
_
U
2H
_
2
/N dU
2H

1
Kittel and Kroemer pg 20
5
2.2 Paramagnetism of a system of N localized spin-1/2 particles
(a) Using expression for (U) calculated in problem 2.1(a) and applying Stirling approximation, nd the relation
between the absolute temperature = kBT and the total energy U of this system.
The temperature is dened by
1
kBT
=
_
(log(g))
U
_
N
(2.2)
The logarithm of 2.1(a) is
log = log N! log
_
N
U
H
2
_
! log
_
N +
U
H
2
_
!
Using Stirlings approximation log N! N log N N
log = N log N N

_ _
HN U
2H
_
log
_
HN U
2H
_

_
HN U
2H
_
+
_
HN +U
2H
_
log
_
HN +U
2H
_

_
HN +U
2H
__
= N log N
_ _
HN U
2H
_
log
_
HN U
2H
_
+
_
HN +U
2H
_
log
_
HN +U
2H
__
Now dierentiating with respect to U
_
(log())
U
_
N
=
1
2H
_
log
_
HN U
H
_
log
_
HN +U
H
__
This means with respect to temperature c
(b) Under what circumstance is negative?
For the temperature to be negative the logarithm has to be negative. The only time that happens, when the term
inside is less than one. This occurs whenever there are more spins against the eld than pointing with it. In that
case, U > 0, so
HN U
HN +U
< 1 HN U < HN +U
(c) Find the magnetization as a function of H and T.
kBT = 2H
_
log
_
HN U
HN +U
__
1
e
2H
k
B
T
=
HN U
HN +U
U
_
1 +e
2H
k
B
T
_
= HN
_
e
2H
k
B
T
1
_
U = HN
e
2H
k
B
T
1
1 +e
2H
k
B
T
U = HN tanh
_
H
kBT
_
The magnetization is dened as the number of magnetic moments per unit volume. Dene n = U/HV
M = (U/HV ) tanh
_
H
kBT
_
ntanh
_
H
kBT
_

6
2.3 Kittel 2.3: Quantum harmonic oscillator
(a) Find the entropy of a set of N oscillators of frequency as a function of the total quantum number n. Use
multiplicity function (1.55)
2
and make the Stirling approximation log N! N log N N. Replace N 1 by N.
Equation 1.55 is
g(N, n) =
(N +n 1)!
(N 1)!(n)!
(2.3)
Replacing N 1 with N in equation 3
g(N, n) =
(N +n)!
(N)!(n)!
The entropy is
= log g (2.4)
Then by Stirling approximation
= (N +n) log(N +n) (N +n) N log N +N nlog n +n
= N log
_
N +n
N
_
+nlog
_
N +n
n
_

(b) Let U denote the total energy n of the oscillators. Express the entropy as (U, N). Show that the total energy
at temperature is
U =
N
e
/
1
(2.5)
The energy is U = n
U

, so the entropy becomes


= N log
_
N +
U

N
_
+
U

log
_
N +
U

_
Then the inverse temperature is
1

=
_
()
U
_
N
=
1

log (N/U + 1)

= log (N/U + 1)
e
/
1 =
N
U
U =
N
e
/
1

2.4 Review Thermal mechanics
Focus on the two stages in the engine cycle of problem 1.4: The isothermal expansion stage followed by the isovolu-
metric stage. The working substance in the engine consist of N helium atoms. What is the change in the multiplicity
of the gas as it traverses the two stages, with the volume increasing from VS(@
h
to VL(@c)?
The entropy for an ideal gas (which helium closely approximates) is given by Equation 3.76, which is
= N
_
log (nQ/n) +
5
2
_
(2.6)
where n N/V and n (m/2
2
)
3/2
. So the multiplicity is
g = e

= e
5N/2
_
nQ
n
_
N
= e
5N/2
_
V
N
_
N _
m
2
2
_
3N/2
2
Kittel and Kroemer pg. 25
7
The multiplicity at VS(@
h
g1 = e
5N/2
_
VS
N
_
N _
m
h
2
2
_
3N/2
The multiplicity at VL(@c
g2 = e
5N/2
_
VL
N
_
N _
mc
2
2
_
3N/2
So the dierence is then
g1 g2 = e
5N/2
_
VS
N
_
N _
m
h
2
2
_
3N/2
e
5N/2
_
VL
N
_
N _
mc
2
2
_
3N/2
=
e
5N/2
N
N
_
m
2
2
_
3N/2
_
V
N
S

3N/2
h
V
N
L

3N/2
c
_

2.5 Review Thermal mechanics
Suppose a body at temperature and pressure P is immersed in a medium that is at the temperature o and pressure
Po, where and pressure P are not necessary equal to o and pressure Po. The body and medium form a closed
system. Now an external source does work to change the state of the body. The source of work is assumed to be
isolated from both the body and the system.
(a) Prove that the minimum work Wmin needed to change the state of the body is given by:
Wmin = (U +PoV +o) (2.7)
The medium can be assume to be large enough that o and Po are constants and then Uo is also constant. The heat
transfer is then
dQo = o doodo = dU +dUo +Wmin +PodVo
Because o is constant and Uo is constant do = dUo = 0. Because the change in the volume of the medium must
equal to change in the volume in the body due to the constant pressure and temperature of the medium, dVo = dV .
So the heat transfer is
odo = dU +Wmin +PodV
According to the second law d
all
0. The change in all entropy is d
all
= d +do. Therefore d do. When
it is at its minimum the two must be equal. With this nal substitution
od = dU +PodV +Wmin Wmin = dU +PodV od
Now to pull the change out front on the right
Wmin = (U +PoV o) (2.8)
(b) Now consider the two special cases
(i) P = Po and = o. What is Wmin equal to in this case.
In this case, U = o PoV + V P Making the substitution in equation 8
Wmin = o PoV + V P +PoV o) = V P = G (2.9)
(ii) P = Po and the process is carried out adiabatically. What is Wmin equal to in this case.
In this case there is no change in the entropy and temperature, U = PoV V P Making the substitution
in equation 8
Wmin = PoV V P +PoV o = o V P = H (2.10)
8
Chapter 3
Problem Set 3
3.1 Kittel 3.2: Magnetic susceptibility
(a) Use the partition function to nd an exact expression for the magnetization M and the susceptibility = dM/dB
as a function of temperature and magnetic eld for the model system of magnetic moments in a magnetic eld.
The result of the magnetization is M = nmtanh(mB/), as derived in (46) by another method. Here n is the
particle concentration.
In this problem there are two dierent energies, with and against the eld. For one particle the partition function is
Z1 = e
mB/
+e
mB/
= 2 cosh
_
mB

_
For N particles the partition function becomes
Z = (Z1)
N
= 2
N
cosh
N
_
mB

_
= 2
N
cosh
N
(mB)
where = 1/. The energy is then
U =
1
Z
Z

U =
1
2
N
cosh
N
(mB)

_
2
N
cosh
N
(mB)
_
=
1
2
N
cosh
N
(mB)
2
N
cosh
N1
(mB) sinh (mB) mB
U = mBtanh (mB) M = nmtanh
_
mB

_
Now for
=
dM
dB
=
d
dB
_
nmtanh
_
mB

__
=
nm
2

sech
2
_
mB

_

(b) Find the free energy and express the result as a function only of and parameter x M/nm.
The free energy is
F = log Z = log (2 cosh (mB/)) = log
_
2 sinh (mB/)
x
_
= log x log (2 sinh (mB/))
(c) Show that the susceptibility is = nm
2
/ in the limit mB
In this case, mB/ 0
=
nm
2

sech
2
_
mB

nm
2

sech
2
0 =
nm
2


9
3.2 Kittel 3.3: Free energy of a harmonic oscillator
A one-dimensional harmonic oscillator has an innite series of equally spaced energy states, with s = s, where s
is a positive integer or zero, and omega is the classical frequency of the oscillator. We have chosen the zero energy
at the state s = 0.
(a) Show that for a harmonic oscillator the free energy is
F = log
_
1 e
/
_
(3.1)
The partition function is
Z =

s=0
e
s/
= 1 +e
/
+e
2/
+. . .
This is a binomial expansion, as long as e
/
1. Then
Z =
1
1 e
/
=
_
1 e
/
_
1
From Eq. 3.55
F = log Z = log
_
1 e
/
_
1
F = log
_
1 e
/
_
In the case, : e
/
1 /. Then
F = log
_
1 e
/
_
log (1 1 + /) F = log (/)
(b) From (87) show that the entropy is
=
/
e
/
1
log
_
1 e
/
_
(3.2)
From Eq. 3.49
=
F

_
log
_
1 e
/
__
=
e
/

2
(1 e
/
)
log
_
1 e
/
_
=
/
e
/
1
log
_
1 e
/
_

3.3 Kittel 3.4: Energy uctuations
Consider a system of xed volume in thermal contact with a reservoir. Show that the mean square uctuation in the
energy of the system is
( ))
2
) =
2
_
U

_
V
(3.3)
Here U is the conventional symbol for ).
_
U

_
V
=
1
Z
2

2
s
e
s/

1
Z
2

2
_

s
se
s/
_
2

2
_
U

_
V
=
1
Z

2
s
e
varepsilons/

1
Z
2
_

s
se
s/
_
2

2
_
U

_
V
=
2
) )
2

10
3.4 Kittel 3.10: Elasticity of polymers
The thermodynamic identity for a one-dimensional system is
d = dU fdl (3.4)
when f is the external force exerted on the line and dl is the extension of the line. By analogy with (32) we form the
derivative to nd

=
_

l
_
U
(3.5)
The direction of the force is opposite to the conventional direction of the pressure.
We consider a polymeric chain of N links each of length , with each link equally likely to be directed to the right
and to the left.
(a) Show that the number of arrangements that give a head-to-tail length of l = 2[s[ is
g(N, s) +g(N, s) =
2N!
_
1
2
N +s
_
!
_
1
2
N s
_
!
(3.6)
The length is given by l = 2[s[ [s[ = l/2. [s[ is the steps of the random walk. So
g(N, s) =
N!
_
N+2s
2
_
!
_
N2s
2
_
!
and g(N, s) =
N!
_
N2s
2
_
!
_
N+2s
2
_
!
The total is the steps to the left and right
g(N, s) +g(N, s) =
2N!
_
N+2s
2
_
!
_
N2s
2
_
!

(b) For [s[ N show that


(l) = log [2g(N, 0)] l
2
/2N
2
(3.7)
Using the Gaussian approximation
g
total
=
2N!
_
N+2s
2
_
!
_
N2s
2
_
!
2g(N, 0)e
2s
2
/N
The entropy is = log g = log (2g(N, 0)) 2s
2
/N, because [s[
2
= l
2
/4
2
. Then
= log (2g(N, 0)) l
2
/2N
2

(c) Show that the force at extension l is


f = l/N
2
(3.8)
From Eq. 3.96
f =
_

l
_
V
=

l
_
log (2g(N, 0)) l
2
/2N
2

f =
l
N
2

11
3.5 Ising spin chain
Based on the prescription of one-dimensional N spin system with only nearest-neighbor interactions J and -J for two
parallel spins and two antiparallel spins, respectively, you are asked to
(a) compute the partition function Z of the spin chain
The partition function
Z =

s
e
s/
=

E
(E)e
E/
If there are N particles then there are N1 pairs. The energy is E = JNpJNa = J(NpNa), where Na = N1Np
E = J(2Np (N 1)). The multiplicity is
=
(N 1)!
Np!(N 1 Np)!

(N 1)!
(N 1 1/2(E/J +N 1))!(1/2(N 1 +E/J))!
The partition function becomes
Z =
J(N1)

E=J(N1)
(N 1)!
(N 1 1/2(E/J +N 1))!(1/2(N 1 +E/J))!
e
E/

(b) determine the entropy of the system at very high and very low temperatures.
The partition function for 1 pair of particles is
Z1 =
_
e
J
+e
J
_
where = 1/. So for N 1 pairs
Z =
_
e
J
+e
J
_
N1
The entropy is given by
= ln Z

Z
Z

When at high , 0, so
= ln Z
0
Z
Z

= ln
_
e
J
+e
J
_
N1
(N 1) ln 2
For low , lets consider a shift in energy from -J to J 0 to 2J. Then
Z =
_
e
J
+e
J
_
N1
Z =
_
1 +e
2J
_
N1
At low , , so
ln Z = ln
_
1 +e
2J
_
N1
ln Z = ln (1 + 0)
N1
ln Z = ln 1 = 0
So
= ln Z = 0
12
Chapter 4
Problem Set 4
4.1 Application of equal partition theorem
Each particle in a system of N particles has a mass m and is free to preform one-dimensional oscillations about its
equilibrium position. Assume the temperature is suciently high so that classical statistical mechanics is relevant.
Calculate the heat capacity of this system of particles in each case of the following restoring forces:
(a) The force is proportional to the square of the particles displacement x from its equilibrium position.
The force is F x
2
i
, so the energy from this force is Ui = hx
3
i
. The total energy is E = (xi) +E

, where (xi) = hx
3
i
.
The average energy is
i =
_

e
E
idx1dx2 . . . dx
f
dp1dp2 . . . dp
f
_

e
E
dx1dx2 . . . dx
f
dp1dp2 . . . dp
f
=
_

i
idxi
_

i
dxi
i =

log
__

i
dxi
_
(4.1)
Now i = hx
3
i
, then
i =

log
__

e
hx
3
i
dxi
_
Let y =
1/3
xi, then
i =

log
__

e
hx
3
i
dxi
_
i =

log
_

1/3
_

e
hy
3
dy
_
=

_
log
1/3
+ log
__

e
hy
3
dy
__
Because this derivative is with respect to only
i =
1
3
i =
1
3

The total energy is then


E = N((pi) +(xi)) E = N
_
1
2
+
1
3

_
E =
5
6
N
So the heat capacity is then
CV =
E

CV =
5
6
N
(b) The force is proportional to [x[
3
13
The force is F x
3
, so the potential energy is U = hx
4
i
, then (xi) = hx
4
i
. From equation 4.1
i =

log
__

e
hx
4
i
dxi
_
Now, letting y =
1/4
xi, then
i =

log
_

1/4
_

e
hy
4
dy
_
=

_
log
1/4
+ log
_

e
hy
4
dy
_
i =
1
4
=
1
4

For the total energy of the system


E = N((pi) +(xi)) E = N
_
1
2
+
1
4

_
E =
3
4
N
So the heat capacity is
CV =
E

CV =
3
4
N
4.2 Thermal Equilibrium of the Sun and Earth
The surface temperature of the sun is To; its radius is R (= 710
8
m) while the radius of the earth is r (= 6.3710
6
m).
The mean distance between the sun and the earth is L (= 1.510
11
m). Assume the earth has reached a steady state
of absorbing and emitting radiation so that its temperature does not change with time.
(a) Find an approximate expression for the temperature T of the earth in terms of the parameters given.
The power of the Sun at its surface is
P = A

T
4

where A

is the surface area of the Sun. The power is constant, so the power at L is P = ALT
4
L
. More important
is the percent that the Earth receives, which is
P =
AeA

AL
T
4

Now at equilibrium this must equal the amount emitted by the Earth
AseT
4
e
=
AeA

AL
T
4

4R
2
e
T
4
e
=
R
2
e
4R
2

4L
2
T
4

T =
_
R

2L
_
1/2
T


(b) Calculate the temperature of the sun given T 290K.
The temperature of the Sun is
T

=
_
2L
R

_
1/2
T
_
2(1.5 10
11
)
7 10
8
290 T

= 6000 K
4.3 Kittel 4.3: Average temperature of the interior of the Sun
(a) Estimate by a dimensional argument or otherwise the order of magnitude of the gravitational self-energy of the
Sun, M

= 2 10
33
g and R

= 7 10
10
cm. The gravitational constant G is 6.6 10
8
dyne cm
2
g
2
.
The self energy is negative. In cgs units energy is ergs=dynecm. The constant has units of dyne
cm
2
g
2
. To remove g
2
,
there needs to be M

squared. For the distance, only the rst power of R

is needed. So the self-energy is


U =
GM
2

= 3.77 10
48
ergs
(b) Assume that the total thermal kinetic energy of the atoms in the Sun is equal to -1/2 the gravitational self-energy.
Estimate the average temperature of the Sun, assuming that there are 1 10
57
particles.
K = U/2
3
2
NkBT = U/2 T =
U
3NkB
T =
GM
2

3NkBR

= 9.1 10
6
K
14
4.4 Kittel 4.6: Pressure of thermal radiation
Show for a photon gas that:
(a)
p =
_
U
V
_

j
sj
_
dj
dV
_
(4.2)
From Eq. 4.39, U =

j
sj)j. Then
p =
_

V
_

j
sjj
__

j
j
sj
V

j
sj
(j)
V

Because sj is the thermal average, it will not change if there is no change in entropy, so
s
j
V
= 0. Then
p =

j
sj
j
V

(b)
dj
dV
=
j
3V
(4.3)
dj
dV
=
_
Nc
L
_
dV
= nc
d(V
1/3
)
dV
=
1
3
nc
V
4/3
=
1
3
nc
LV
=
1
3
j
V

(c)
p =
U
3V
(4.4)
From parts a and b
p =

j
sj
dj
dV
=

j
sj
_

1
3
j
V
_
p =
1
3
U
V

(d) Compare the pressure of the thermal radiation with the kinetic pressure of gas of H atoms at a concentration
of 1 mole cm
3
. At what temperature (roughly) are the two pressure equal?
For the hydrogen gas p =
nRT
V
, for photons p =

2
k
4
B
45
3
c
3
T
4
. Then
nRT
V
=

2
k
4
B
45
3
c
3
T
4
T =
_
45
3
c
3
nR

2
k
4
B
V
_
1/3
T = 3.21 10
7
K
4.5 Kittel 4.7: Free energy of a photon gas
(a) Show that the partition function of a photon gas given by
Z = n
_
1 e
n/
_
1
(4.5)
where the product is over the modes n.
For one mode Z1 =
1
1e

1
/
. For the i
th
mode, Zi =
1
1e

i
/
. The total partition function is given by
Z = Z1 . . . Zi . . . =
_
1
1 e

1
/
_
. . .
_
1
1 e

i
/
_
. . . = n
_
1
1 e
n/
_
Z = n
_
1 e
n/
_

15
(b) The Helmholtz free energy is found directly from above as
F =

n
log
_
1 e
n/
_
(4.6)
Transform the sum to an integral to nd
T =

2
V
4
45
3
c
3
(4.7)
F = log Z = log n
_
1 e
n/
_
1
= log n
_
1 e
n/
_
F =

n
log
_
1 e
n/
_
From Eq. 4.17
F =

n
log
_
1 e
n/
_
=
1
8
_

0
4n
2
log
_
1 e
nc/L
_
=
2
8
_

0
4n
2
dn
nc
L
e
nc/L
1 e
nc/L
Letting x =
nc
L
and Eq. 4.18 and 4.19
F =

_
L
c
_
3
_

0
x
3
dx
e
x
1
=

_
L
c
_
3

4
15
=

2
L
3

4
45
3
c
3
F =

2
V
4
45
3
c
3

4.6 Kittel 4.18: Isentropic expansion of photon gas
Consider the gas of photons of the thermal equilibrium radiation in a cube of volume V at temperature . Let
the cavity volume increase; the radiation pressure performs work during the expansion, and the temperature of the
radiation will drop. From the results for the entropy we know that V
1/3
is constant in such an expansion.
(a) Assume that the temperature of the cosmic black-body radiation was decoupled from the temperature of the
matter when both were at 3000 K. What was the radius of the universe at that time, compared to now? If
the radius has increased linearly with time, at what time, at fraction of the present age of the universe did the
decoupling take place?
For the universe R = ct and V
1/3
= constant, where R is the radius t is the time, and c is some constant. Then
V
1/3
R = constant. Then
iRi =
f
R
f

Ri
R
f
=

f
i
So
R
i
R
f
= 2.9/3000 1/1000 so the radius was about 1/1000 the present radius. Because the growth is linear c =
R
i
t
i
.
Then
Ri
ti
t
f
= R
f

Ri
R
f
=
t
f
ti
=
1
1000
present time
(b) Show that the work done by the photons during the expansion is
W =
_

2
15
3
c
3
_
Vi
3
i
(i
f
) (4.8)
where the subscripts i and f refer to the initial and nal states.
dW = pdV dW =
1
3
U
V
dV dW =

2

4
45
3
c
3
dV
By V
1/3
= constant
3
V = constant. Now

3
dV = 3
2
V d dV = 3
1
V d
Remembering
3
V is a constant, so
3
V =
3
i
Vi. The work is then
W =
_

f

3
i
Vi
15
3
c
3
d W =

2

3
i
Vi
15
3
c
3
(i
f
)
16
Chapter 5
Problem Set 5
5.1 5.1: Kittel 4.14: Heat capacity of liquid
4
He at low temperature
5.2 5.2: Kittel 4.15: Angular distribution of radiant energy ux
5.3 5.3: Qualifying exam le problem
5.4 5.4: Helmholtz free energy for the Debye model
5.5 5.5: Kittel 5.3: Potential energy of gas in gravitational eld
17
18
Chapter 6
Problem Set 6
6.1 6.1: Kittel 5.4: Active transport
Consider the concentration of K
+
is 10
4
inside the plant cell compared to the pond. Find the chemical potential
dierence.
Treating the ions as an ideal gas,the chemical potential dierence is
= in out = log
_
nin
nQ
_
log
_
nout
nQ
_
= log
_
nin
nout
_
Inserting the values
=
300 K
11605 K/eV
log(1000) 0.24 eV
6.2 6.2: Kittel 5.7: States of positive and negative ionization
For the following system,
State # of electrons Energy
Ground 1 /2
Positive ions 0 /2
Negative ions 2 /2
Excited 1 /2
Find the condition for

N
_
= 1.
The grand partition function is
: = e
/2
+e
/2
+
2
e
/2
+e
/2
(6.1)
So the average number is

N
_
=
1
:

ASN
N
N
e
s/

N
_
=

_
e
/2
+e
/2
_
+ 2
2
e
/2
(e
/2
+e
/2
) +
2
e
/2
+e
/2
(6.2)
For

N
_
= 1, then = e
/2

19
6.3 6.3: Kittel 5.10: Concentration uctuation
(a) Show that

N
2
_
=

2
:

2
:

2
(6.3)
Now

N
2
_
=

ASN
N
2
e
(Ns)/
:
=

2
:

ASN

2
e
(Ns)/
=

2
:

ASN
e
(Ns)/

N
2
_
=

2
:

2
:

2

(b) Show that

(N)
2
_
=

N
_

(6.4)
Beginning with

N
_

_

:
_
:

__
=
2
_
1
:

2
:


1
:
2
:

_
which reduces to

2
_
1
:

2
:

2

1
:
2
_
:

_
2
_
=
_
N

N
__
2
_
=

(N)
2
_

6.4 6.4: Kittel 6.3: Distribution function for double occupancy statistics
For a new system, which has orbitals 0,1,2 and energies 0,,2, respectively
(a) Derive

N
_
, so
: = 1 +e
/
+
2
e
2/
(6.5)
So

N
_

N
_
=
1
:
_
0 0 + 1 e
/
+ 2
2
e
2/
_

N
_
=
e
/
+ 2
2
e
2/
1 +e
/
+
2
e
2/
(6.6)
(b) Now if is double generate
The partition function is now
: = 1 + 2e
/
+
2
e
2/
=
_
1 +e
/
_
2
(6.7)
Then

N
_
=
1
:
_
0 0 + 2 e
/
+ 2
2
e
2/
_
=
2e
/
_
1 +e
/
_
(1 +e
/
)
2
=
2e
/
1 +e
/
Now

N
_
=
2

1
e
/
+ 1

N
_
=
2
e
()/
+ 1
(6.8)
20
6.5 6.5: Kittel 6.7: Relation of pressure and energy density
(a) Show that
p =

s
_
s
V
_
N
e
s/
Z
(6.9)
So beginning with the partition function, Z =

s
e
s/
, then the Helmholtz free energy is
F = log Z = log
_

s
e
s/
_
so the pressure is then
p =
F
V

,N
=

Z
V
Z
p =

s
_
s
V
_
N
e
s/
Z

(b) Show that for the free particles
_
s
V
_
N
=
2
3
s
V
(6.10)
for the boundary conditions of the system
The energy levels for these boundary conditions
_
s
V
_
N
=

V
_

2
n
2
2m
V
2/3
_
=
2
3
_

2
n
2
2m
V
5/3
_
=
2
3
s
V

(c) Show that
p =
2U
3V
(6.11)
From part 1 and 2
p =

s
_
s
V
_
N
e
s/
Z
=

s
_
2
3V
s
_
e
s/
Z
Then using the denition of the partition function and energies
p =
2
3V
_

s
se
s/
Z
_
p =
2U
3V

6.6 6.6: Kittel 6.9: Gas of atoms with internal degree of freedom
For an ideal gas with an internal degree of freedom, . The internal partition is for one particle is Zint =
e
/
_
1 +e
/
_
(a) Find the chemical potential
Because the logarithm is being taken of the partition function the internal and ideal gas can be separated. This is in
fact true for all of this problem. From Equation 6.48 = [log (n/nQ) log Zint]
= [log(n/nQ) log Zint]
=
_
log(n/nQ) +

log(1 +e
/
)
_

21
(b) Free energy
From Equation 6.49, Fint = N log Zint
F = N (log (n/nQ) 1) N log
_
e
/
_
1 +e
/
__
= N (log (n/nQ) 1) +N N log
_
1 +e
/
_

(c) Entropy
From Equation 6.50, int = (F/)
V
=
ideal
+int
= N [log(n/nQ) + 5/2]

_
N N log
_
1 +e
/
__
= N [log(n/nQ) + 5/2] +N log
_
1 +e
/
_
+
N
(e
/
+ 1)

(d) Pressure
Notice that the internal partition function is independent of the volume. From Equation 3.49 p =
_
F
V
_

, then
pint = 0, so the pressure is just the ideal gas, which is p =
N
V

(d) Heat capacity at constant pressure.
For an ideal gas, Cp =
5
2
N. Now from Equation 6.37, but the volume is constant, so
Cp =
_
U

_
p
+p
_
V

_
p
Cp =
_
U

_
p
Now the energy needs to be found. So the relation Uint = Fint +int yields the internal energy.
Uint = N N log
_
1 +e
/
_
+N log
_
1 +e
/
_
+
N
e
/
+ 1
=
N
e
/
+ 1
So the heat capacity for the internal energy is
Cp =

N
e
/
+ 1
=
N
2

2
e
/
(e
/
+ 1)
2
The total heat capacity at constant pressure
Cp =
5
2
N +
N
2

2
e
/
(e
/
+ 1)
2

22
Chapter 7
Problem Set 7
7.1 7.1: Pressure in types of gases
7.2 7.2: Kittel 5.13: Isentropic expansion
7.3 7.3: Kittel 6.8: Time for a large uctuation
7.4 7.4: Kittel 6.10: Isentropic relations of ideal gas
7.5 7.5: Kittel 6.12: Ideal gas in two dimensions
7.6 7.6: Kittel 7.4: Chemical potential versus temperature
7.7 7.7: The absorbtion of gas onto a surface
23
24
Chapter 8
Problem Set 8
8.1 8.1: Mixing of two distinct atoms
Calculate the entropy change when an impermeable partition separating two compartments each of volume V and
each containing an ideal monatomic gas of N identical atoms at temperature is removed. However, the atoms in
one compartment are distinguishable from those in the other compartment.
To identify each set of particles one side will be A and the other B. The change in entropy for A particles is given
by 2 1 = N log
_
V
2
V
1
_
, but V2 = 2V1, so the change in entropy of A is then A = N log 2. This is identical for
B because there is no description of A in nding of the change in the entropy. So the change in the entropy for B is
B = N log 2. The total change in entropy is then A + B = = 2N log 2
8.2 8.2: Kittel 7.3: Pressure and entropy of degenerate Fermi gas
(a) Show that a Fermi electron gas in the ground state exerts a pressure
p =
_
3
2
_
2/3
5

2
m
_
N
V
_
5/3
(8.1)
From earlier in Kittel, the pressure is
p =
_
U
V
_

p =

V
_
3
5
NF
_
p =
3
5
N
F
V
Now expanding the Fermi energy
p =
3
5
N
F
V
p =

2
2m
3
5
N

V
_
_
3
2
N
V
_
2/3
_
=
3
5

2
N
5/3
2m
_
3
2
_
2/3
V
_
1
V
2/3
_
p =

2
N
5/3
5m
_
3
2
_
2/3
V
5/3
p =
_
3
2
_
2/3
5

2
m
_
N
V
_
5/3

(b) Find an expression for the entropy of a Fermi electron gas in the region F
The heat capacity of a Fermi gas is CV =
1
2

2
N/F . The heat capacity is related to the entropy by
CV =
_

_
V
=
CV


Using the heat capacity of Fermi gas
d =
_

0
1
2

2
N/ (F ) d =
1
2

2
N/F
25
8.3 8.3: Kittel 7.6: Mass-radius relationship for white dwarfs
Consider a white dwarf of mass M and radius R. Let the electrons be degenerate but nonrelativistic; the protons are
nondegenerate.
(a) Show that to the order of magnitude of the gravitational self-energy is GM
2
/R, where G is the gravitational
constant.
The density for an uniform sphere is =
3M
4R
3
. So the potential self-energy is
V =
Gmm(r)
r
=
G4r
2
rm(r)
r
=
G4rr
2
4r
3
3
=
16G
2

2
r
4
r
3
=
3GM
2
R
6
r
4
r
Then the total potential self-energy is
V =
_
R
0
3GM
2
R
6
r
4
dr V =
3GM
2
5R

GM
2
R

(b) Show that the order of magnitude of the kinetic energy of the electrons in the state with mass of an electron,
m, and mass of a proton, MH, is

2
N
5/3
mR
2


2
M
5/3
mM
5/3
H
R
2
(8.2)
The energy of one electron is =

2
2m
_
3
2
N
V
_
2/3
. There are N electrons in the Sun. The total energy would be then
K =

2
N
2m
_
3
2
N
V
_
2/3
. The volume of the Sun is V =
4
3
R
3
. The energy becomes
K =

2
2m
_
9N
4R
3
_
2/3
N =

2
N
5/3
mR
2
_
1
2
_
9
4
_
2/3
_
Because the mass of the proton is so much more than that of the electrons, so then all that has to be considered is
mass of the proton. So the number of particles are N M/MH. The terms in [. . .] are on the order of two, so to the
order of magnitude, the kinetic energy is
K =

2
N
5/3
mR
2
_
1
2
_
9
4
_
2/3
_


2
mR
2
_
M
MH
_
5/3

(c) Show that if the self-energy and kinetic energy are equal, that M
1/3
R 10
20
g
1/3
cm.
So immediately, then
GM
2
R
=

2
mR
2
_
M
MH
_
5/3
M
1/3
R =

2
mGM
5/3
H
M
1/3
R
_
10
27
_
2
10
8
10
27
(10
24
)
5/3
= 10
20
g
1/3
cm
(d) What is the density of a white dwarf with its mass equal to the mass of the Sun (2 10
33
g)
The density would be
M
1/3
R = 10
20

_
MR
3
_
1
= 10
60
=
3M
2
410
60
=
3
_
2 10
33
_
2
410
60
= 955000 g/cm
3

26
(e) Show for a neutron star M
1/3
R 10
17
g
1/3
cm. What is the radius of the neutron star whose mass is equal to
that of the Sun?
Looking back at part c, M
1/3
R =

2
mGM
5/3
H
, but the mass of a neutron is Mn 1000m and that MH Mn. Then
M
1/3
R =

2
MnGM
5/3
H
=

2
1000mGM
5/3
H
10
17
g
1/3
cm
The radius of a neutron star with mass of the Sun would be
M
1/3
R = 10
17
g
1/3
cm
_
2 10
33
_
1/3
R = 10
17
g
1/3
cm
R = 794000cm R = 7.94km
8.4 8.4: Kittel 7.10: Relativistic white dwarfs stars
Consider a Fermi gas of N electrons each of rest mass m in a sphere of radius R. Treating the electrons as relativistic
for a white dwarf, nd the following
(a) Using the viral theorem nd the value of N, neglecting the kinetic energy of the protons.
From problem 7.2, the kinetic energy for the electrons is K +
3
4
NF and the Fermi energy is F = c (3N/V )
1/3
.
Using the volume of the sphere, the kinetic energy becomes K =
3c
4R
_
9
4
2
_
1/3
N
4/3
. The potential energy is from
Problem 7.6, V =
3GM
2
5R

3GN
2
M
2
H
5R
. Applying the viral theorem
3GN
2
M
2
H
5R
=
3
4
c
R
_
9
4
2
_
N
4/3
N
2/3
=
5c
4GM
2
p
_
9
4
2
_
1/3
N =
_
5c
4GM
2
p
_
9
4
2
_
1/3
_
3/2

(b) Estimate the value of N


Inserting the values
N =
_
5
4
3 10
8
_
1.05 10
34
_
6.6 10
11
(1.67 10
27
)
2
_
9
4
2
_
1/3
_
3/2
= 8.14 10
57

8.5 8.5: Electrons in the air o a conductor


The lowest possible energy of a conduction electron in a metal is Vo below the energy of a free electron at innity.
The conduction electrons have a chemical potential energy . The minimum energy needed to remove an electron
from the metal is = Vo and is called the work function of the metal. Consider an electron gas outside the metal
in thermal equilibrium with the electrons in the metal at the temperature . Assume , nd the mean number
of electrons per unit volume outside the metal.
The mean number of electrons per unit volume outside the metal can be found by relating the chemical potentials.
The chemical potential inside the conductor is . Outside the conductor, the chemical potential is
ideal
+ + .
The chemical potential for an ideal gas is = log (n/nQ). Therefore

ideal
+ + = log (n/nQ) + + = n = nQe
/
27
Now n = N/V , and nQ =
_
m
2
2
_
3/2
. Then
N/V =
_
m
2
2
_
3/2
e
/
Lastly, there is a spin 2 degeneracy, so
N
V
= 2
_
m
2
2
_
3/2
e
/

28
Chapter 9
Special Problem Set (9)
9.1 Problem 1: Properties of Photon Gas
The Helmholtz free energy of a certain system is given by F = bV
4
, where b is a constant and all other symbols
have their usual meaning.
(a) Compute the chemical potential of the system.
The chemical potential is related to the free energy by
=
_
F
N
_
,V
=

N
bV
4
= 0
(b) What is the energy of the system?
The energy of the system is related to the free energy by
U =
2
(F/)

=
2

bV
3
U = 3bV
4

(c) Find the equation of state that describes the system.


The equation of state is related to the pressure, so
p =
_
F
V
_

p = b
4
or p =
U
3V

(d) What is the work done during an isentropic expansion from 1 to 2?
So rst the entropy has to be found = (F/)V = 4bV
3
.
dW = pdV
1.c
dW =
1
3
U
V
dV
1.b
dW = b
4
dV
An isentropic expansion =constant, so
3
V =constant. Then dV = 3
1
V d. The work is then dW = 3bV
3
d,
but because the expansion is isentropic V
3
=constant. With that choose it to be the initial state 1. Finally
W =

2 _

1
3bV1
3
1
d W = 3bV1
3
1
(2 1)
9.2 Problem 2: Engine Cycle
For an ideal-gas engine cycle the processes AB and CD are isentropic, process BC is isobaric, and process DA is
isovolumetric.
(a) Calculate the engine eciency in terms of the temperatures A, B, C, D and (= Cp/CV ).
29
The eciency is =
Q
h
Q
l
Q
h
. So the heat comes and leaves the system in only two places, because AB and CD are
isentropic. The heat in is Q
h
= Cp (C B). The heat out is Q
l
= CV (D A). So the eciency is
=
Q
h
Q
l
Q
h
=
Cp (C B) CV (D A)
Cp (C B)
= 1
D A
(C B)

(b) Find /C, where C is the eciency of a Carnot engine operating at the highest and lowest temperatures of
the cycle.
First nd the highest and lowest temperatures. From the graphs and dened process: VCB = VBC and VC > VB
C > B; pDA = pAD and pD > pA D > A; and AV
1
A
= BV
1
B
and VA > VB B > A. The Carnot
eciency is C = 1

A

C
. Also from the ideal gas law, B =
VB
VC
C . D =
_
V
C
V
D
_
1
C. V
1
D
=
V

B
V
C

A
. Combining
the statements D =
_
VC
VB
_

A The eciency is then


= 1
A
__
V
C
V
B
_

1
_
C (1 VB/VC)
Then the ratio is

C
=
1

A
__
V
C
V
B
_

1
_

C
(1V
B
/V
C
)
1

A

(c) Is /C> 1 or < 1.


From previous knowledge, it should be /C < 1. Now the check
1

A
__
V
C
V
B
_

1
_

C
(1V
B
/V
C
)
1

A

C
< 1
A
__
V
C
V
B
_

1
_
C (1 VB/VC)
>
A
C
(VC/VB)

1 > (1 VB/VC)
This statement is true as long as > 1, which is true by the fact that = Cp/CV = (CV + 1)/CV > 1.
9.3 Problem 3: Vibrational Modes of a Molecule
Suppose that in a certain polyatomic ideal gas, the vibrational motions of each molecule can be described in terms
of three independent simple harmonic oscillators having the same frequency /2. The vibrational energy of each
molecule is then given by n =
_
na +n
b
+nc +
3
2
_
, where n = na +n
b
+nc with all the ns being 0, 1, 2, . . .
(a) What is the degeneracy of the vibrational states of each molecule as a function of n?
The degeneracy for S independent harmonic oscillators is (S+n1)!/(S1)!(n)!. Here S = 3 then g(n) =
(n + 2)!
n!2

(b) Write down an expression to represent the vibrational part of the molecular partition function (call it Z
vib
).
The partition function is the sum of the Boltzmann factor. So Z
vib
=

n=0
1
2
_
n
2
+ 3n + 2
_
e
3/2
e
n/

(c) Calculate Z
vib
at high temperature, i.e. assume

1.
30
The partition function at high temperature
Z
vib
=
e
3/2
2
_

0
_
n
2
+ 3n + 2
_
e
n/
dn
=
e
3/2
2
_
(3)
(/)
3
+
3(2)
(/)
2
+
2(1)
/
_
Now applying
Z
vib
=
_
1
3
2

__
3
3
()
3
+
3
2
()
2
+

_
(d) Evaluate the heat capacity and energy per molecule at high temperature.
The energy is U
vib
=
2 log Z
vib

, which is
U
vib
=
2

log
__
1
3
2

__
3
3
()
3
+
3
2
()
2
+

__
=
2
_
18
2
6 z
(2 3) (3
2
+ 3 + 1)
_

U
vib
=
2
18
2
6
3
The total energy per molecule is Utot = U
ideal
+ U
vib
Utot =
3
2
+ 3 Utot =
9
2
. The heat capacity is
CV =
U

CV =

9
2
CV =
9
2

9.4 Problem 4: Relativistic Massless Bosons
Consider an ideal gas of massless bosons in thermal equilibrium. The number of such bosons in the system is constant.
(a) Derive an expression for the number of thermally excited bosons above the ground state if the energy of the
bosons happens to exhibit the dispersion relation = ck.
First k is the wavenumber or k = n/L. The partition function would be Z =

n=0
e
n/L
. Then
n) = Z
1

ne
n/L
n) =
1
e
/L
1
Then
Ne =

n) =
1
8
_

0
4n
2
dn
e
n/L
1
=
V
3
2
2

3
c
3
_

0
x
2
dx
e
x
1
Ne = 2(3)
V
3

3
c
3
Ne 1.202
V
3

3
c
3

(b) Find the critical temperature if the number density of 10
20
cm
3
below which Bose-Einstein condensation (BEC)
occur.
For a BEC, N = Ne. Then
10
20
V = 1.202
V
3

3
c
3
= c
3
_
10
20

2
1.202
T = 21458 K
31
9.5 Problem 5: Kittel & Kroemer 7.9 and more
(a) Calculate the integral for Ne() for a one-dimensional gas of noninteracting bosons, and show that the integral
does not form in one dimension. Take = 1 for the calculation.
From Equation 7.86, T() =
L

_
2m

_
1/2
. Then
Ne =
_
dT()f(, ) =
_

0
d
L

_
2m

_
1/2
1
e
/
1
=
L

_
2m

2
_
1/2

1/2
_

0
x
1/2
dx
e
x
1
Taking the rst order term on the exponent
_

0
x
1/2
dx/(1 +x 1) = 1/2x
1/2

0
, which does not converge. Any
higher order terms will diverge also.
(b) What functional form does the density of state (DOS) need to have for an abrupt transition to occur in a 1-D
and 2-D Boson gases? What is the range of in 1-D? In 2-D?
32
Chapter 10
Problem Set 10
10.1 10.1: Collisions with a wall for a Fermi Gas
Calculate the average rate of collision per unit area with a wall in a Fermi gas at T = 0, which will be called .
Lets think of this problem in momentum space. With T = 0, f = 1 of the fermi-dirac distribution. For an isotropic
sphere in momentum space: sin ddp
2
dp/(2)
3
. Then the velocity is v n = v cos . Remembering that v = p/m.
So
=
p
F _
0
/2
_
0
2
_
0
(2s + 1)
(2)
3
p
3
m
cos sin dddp
=
(2s + 1)
m(2)
3
p
F _
0
p
3
dp =
(2s + 1)
4(2)
3
p
4
F
m
Now, pF = 2mF , and F =

2
2m
_
6
2
N
(2s+1)V
_
2/3
=
(2s + 1)
4(2)
3
4m
2
m
_

2
2m
_
2
_
6
2
N
(2s + 1)V
_
4/3
=
3(6
2
)
1/3
16(2s + 1)
1/3

m
_
N
V
_
4/3

10.2 10.2: Free energy and pressure of a Boson gas


Determine the free energy F and pressure P on a Boson gas at E. Include the degeneracy factor (2s + 1) in
your answers.
From the class notes U = NeI, where Ne =
1.306V
4
_
2M

2
_
3/2

5/2
(2s + 1) and I = .7702. Now U =
2
(F/)/.
Then
F/ =
_

1

2
Ud
F/ =
.7702(1.306)V
4
_
2M

2
_
3/2
(2S + 1)

_
0

1/2
d
F =
.7702(1.306)V
4
_
2M

2
_
3/2
2
3
(2s + 1)
5/2
F = .4742V (2s + 1)
_
M

2
_
3/2

5/2
=
2
3
U
33
Now for the pressure
p =
_
F
V
_

p =
2(.7702)(1.306)
(3)4
_
2M

2
_
3/2
(2s + 1)
5/2
p = .4742(2s + 1)
_
M

2
_

5/2
=
2
3
U
V

10.3 10.3: Discontinuity in the slope of the heat capacity of a Bose gas
Provide the missing steps on the derivation of the heat capacity of a Bose gas near E. That is, use U expression
to show that CV = 0 and
_
C
V

_
=
3.66N

E
at = E
From the lecture notes
Uex Uo = U =
3
2

6
m
3
N=0
_
N=0 N
V
_
2
Now N=0 = N
_

E
_
3/2
. So at = E N=0 = N. It is obvious that the squared term is 0 from N N = 0,
then U = 0. Now CV =
U

, and CV =
(Uo+U)


Uo

=
U

= CV . Which is
CV =
3
2

6
m
3
V
2
_
N=0

_
N=0 N

_
2
+ 2N=0
_
N=0 N

__
1

N=0


N=0 N

2
__
Every term is multiplied by N=0 N, which is 0 at = E. Now for
C
V

, which is

CV

=

2
(Uo + U)

2


2
Uo

2
=
CV

The only nonzero term is the second term by the same argument as before.
C
V

=
6
2

6
m
3
V
2

N
=0

2

N
=0
N

+N
=0

2
N
=0

N
=0
N

+N
=0
1

N
=0

2N
=0
N
=0

N
=0
N

Again, by N=0 N = 0, only one term remains


CV

=
6
2

6
m
3
V
2
N=0
_
1

N=0

_
2
Now using N=0 = N(/E)
3/2
. Then
CV

=
6
2

6
m
3
V
2
N
_

E
_
3/2
_
3
2
N

1/2

3/2
E
_
2

CV

=
6
2

6
m
3
V
2
9
4
N
3

3
E

=
E
CV

=
6
2

6
m
3
V
2
_
9
4
_
N
2
_
m
2
2
_
3
_
2.612V
N
_
2
N
E
= 3.66
N
E

10.4 10.4: Maximum work extracted from an ideal gas


Determine the maximum work that can be extracted from an ideal gas system held at constant volume by cooling it
from temperature 1 to a temperature o of the medium.
The amount of heat that the system gives o can be turned into work is
Qgain =

1 _
o
NCV d Q = NCV (1 o)
34
But there is some energy lost to the change in entropy
=

1 _
o
NCV

d = NCV log
_
1
o
_
The heat is Q
lost
= = NCV o log
_

1
o
_
. The dierence of the two is the maximum work that can be done by
the ideal gas
W = NCV
_
1 o o log
_
1
o
__

35
36
Chapter 11
Problem Set 11
11.1 11.1:A review problem
The entropy of a monatomic ideal gas can be written as = (3N/2) ln + terms that involve N, V , M and some
constants. By including the internal degrees of freedom in the entropy of a polyatomic ideal gas, prove that the factor
3N/2 must be replaced by the factor
N
1
.
From problem 11.3,
CV =
_

_
V
d =
CV

d = CV ln
Now
CV =
_

_
V
= Cp +
_

p
_

_
p

_
V
= Cp
_
V

_
p
_
p

_
V
Now applying the ideal gas law pV = N
Cp
_
V

_
p
_
p

_
V
= Cp
N
p
N
V
= Cp N
Then CV = Cp N.
N
1
=
CV N
Cp CV
=
CV N
N
= CV
N
1
= CV
Then from the rst equation
= CV ln =
N
1
ln
11.2 11.2: Dissociation of water
A certain number of moles of H2O are introduced into a container of volume V . At some high temperature T
dissociation takes place according to 2H2O 2H2 + O2. Let x denote the fraction of water vapor molecules which
are dissociated at T corresponding to a total pressure P.
(a) Derive an expression that relates x to p and a constant that depends only on temperature.
The partial pressure is pi = ni, so the total pressure is
p =

i
ni
Now for each water molecule that dissociates, there are 3/2 molecules. Then
p = (1 x)n +
3
2
xn =
_
1 +
x
2
_
n
37
Then from class
N
2
H
2
O
N
2
H
2
HO
2
= KN()
Factoring the concentration n = N/V
(1 x)
2
n
2
(x
2
)
x
2
n
3
= Kn()
2(1 x)
2
x
3
Kn()
= n
Placing that into the pressure
p =
2(1 +x/2)(1 x)
2
x
3

Kn()

(b) Write down an equation that relates the chemical potentials of H2O, H2 and O2 in equilibrium.
In equilibrium, the Gibbs free energy is 0.
G = iNi = 0
Now, rewriting the chemical equation as
2H2O + 2H2 +O2 = 0 2NH
2
O + 2NH
2
+NO
2
Combining the two equations
iNi = 2H
2
ONH
2
O + 2H
2
NH
2
+O
2
NO
2
= 0
Setting the number equal as part of the equilibrium conditions
2
H
2
O
N
H
2
O
+ 2
H
2
N
H
2
+
O
2
N
O
2
= 0
_
2
H
2
O
+ 2
H
2
+
O
2
_
N = 0
H
2
+
1
2

O
2
=
H
2
O

11.3 11.3: Practice with the Jacobian
Since any thermodynamical quantity of a system with a xed number of particles can be written as a function of
two variables that are not necessarily the same as those used in class, thus you may select the appropriate functional
forms to derive the following expressions:
_
U

_
V
=
_

_
V
_
U
V
_

= p +
_

V
_

Express the Helmholtz free energy F as a function of T and V to prove that


_

V
_

=
_
p

_
V
so that one of the above
expressions can be also be written as
_
U
V
_

= p +
_
p

_
V
Writing the energy in terms of entropy and volume
U(, V ) dU = d pdV
dU =
_

_
V
d +
_

_

V
_

p
_
dV
Now for the relationship between the energy and temperature holding the volume constant
_
U

_
V
=
_

_
V

38
Then the relationship between the energy and volume holding the temperature constant
_
U
V
_

= p +
_

V
_

Writing the free energy in terms of temperature and volume


dU = d() d pdV dF = d +pdV
Then
dF =
_
F

_
V
d +
_
F
V
_

dV
_
F

_
V
= ;
_
F
V
_

= p

2
F
V
=

2
F
V

_

V
_

=
_
p

_
V

11.4 11.4: More practice


Prove that
_
C
V
V
_

=
_

2
p

2
_
V
From problem 11.3
_
CV
V
_

=

V
_

_
=

2

V
=
_

_
V
_

V
_

Again from problem 11.3

_
V
_

V
_

=
_

_
V
_
p

_
V
=
_

2
p

2
_
V

11.5 11.5: van der Waals Gas


Use some of the results from problems 11.3 and 11.4 above to obtain a formula for the energy of a van der Waals
gas which obeys the equation of state
_
p +a/V
2
_
(V b) = (R/kB) , where R is the gas constant. Note that your
answer will also involve the specic heat function CV .
The pressure is
p =
R
kB

V b

a
V
2
From problem 11.3
dU = C
V
d +
_

_
p

_
V
p
_
dV U

+dU

= C
V
+
_

_
p

_
V
p
_
dV
Then with the pressure from above

_
p

_
V
p =
R
kB

V b

R
kB

V b
+
a
V
2
=
a
V
2
_
a
V
2
dV =
a
V
Placing this in the second equation
U = CV
a
V
+ constant
39
40
Chapter 12
Problem Set 12
12.1 12.1: Maxwell Relations
The formula Cp CV =
_

p
_

_
p

_
V
was derived in class
(a) Show that Cp CV = V K
_
p

_
2
V
, where K =
1
V
_
V
p
_

is isothermal compressibility.
So what needs to be shown here is that
_

p
_

=
_
V
p
_

_
p

_
V
. Now using the Jacobian and that
_
p

_
V
=
_

V
_

p
_

=
_
V
p
_

_

V
_
V
=
(V, )
(p, )
(, )
(V, )
=
(, )
(p, )
=
_

p
_

From this
Cp C
V
=
_

p
_

_
p

_
V
=
_
V
p
_

_

V
_
V
_
p

_
V
=
V
V
_
V
p
_

_
p

_
V
_
p

_
V
Cp C
V
= V K
_
p

_
2
V

(b) If the entropy (p, T) = a(p)


n
at the low temperature, where a(p) is a function of pressure, show that CpCV =
1
V K
_
da
dp
_
2

2n+1
. This result implies that there is no distinction between Cp and CV at very low temperatures.
From the rst part,
1
K
= V
_
p
V
_

. Matching the terms, it needs to be shown that


_
p
V
_

p
_

=
_
p

_
V
. Then
_
p

_
V
=
_
p
V
_

_

p
_

=
(p, )
(V, )
(, )
(p, )
=
(, )
(V, )
=
_

V
_

From the rst part


_

V
_

=
_
p

_
V
. So
_
p
V
_

p
_

=
_
p

_
V
, Placing this into the original equation with the
denition of K .
Cp CV =
_

p
_

_
p

_
V
=
1
V K
_

p
_
2

Now (p, T) = a(p)


n
, so
1
V K
_
(a(p)
n
)
p
_
2
Cp CV =
1
V K
_
da
dp
_
2

2n+1

12.2 12.2: Equilibrium conditions


A substance is in equilibrium in the presence of externally applied forces. A small number of particles or a small
amount of energy, or both, are allowed to transfer between any two innitesimal volume elements of this substance.
Prove that the temperature and chemical potential must each have a constant value.
From the rst law of thermodynamics, W = U Q. The system is at equilibrium so W = U = 0. Now lets
consider two subsystems 1 and 2, then U1 = U2 = 0, Q1 = Q2. Now from the denition of heat, (11) =
(22). From equilibrium terms tot is constant, so are any smaller sections. Then 11 = 22 1 = 2. The
only way this can be true at equilibrium is if 1 = 0 = 2. Now the total number of particles is constant. This
means (1N1) = (2N2) 1N1 = 2N2 1 = 2 or that the chemical potential is constant.
41
12.3 12.3: Fluctuation in number of a Fermi gas
Determine (N)
2
for an electron gas at temperatures much lower than the Fermi temperature.
From lecture notes
(N)
2
=
_
N

_
,V
At temperatures below the Fermi temperature, the number is
N =
V
3
2
_
2mF

2
_
3/2
Then
(N)
2
=
_
N

_
,V
=

_
V
3
2
_
2m
F

2
_
3/2
_
=
V
2
2
_
2m

2
_
3/2

1/2
F
=
V
2
2
_
2m

2
_
3/2
_

2
2m
_
1/2
_
3
2
N
V
_
1/3
(N)
2
=
3
1/3
mV

4/3

2
_
N
V
_
1/3

12.4 12.4: Fluctuation in volume, pressure, entropy and temperature


Find
(a) V p
From class
V p =
C
V

2
+
_

V
_

V
_
p
V
_
V
V
_
p
V
_

V
2
=
C
V

_
p
V
_

V
2
By the fact that we are looking for function of p and V means we need V p) =
_
p
V
_

V
2
). From the class
notes V
2
) =
_
V
p
_

. Substituting this into equation above


V p) =
_
p
V
_

_
V
p
_

=
(b)
From the previous problem, ) =
C
V


2
). From class,
2
) =

2
C
V
. Combining these two
) =
CV


2
) =
CV

2
CV
=
12.5 12.5: Kittel 10.5: Gas-solid equilibrium
Consider the gas-solid equilibrium under the extreme assumption that the entropy of the solid may be neglected over
the temperature range of interest. Let o be the cohesive energy of the solid, per atom. Treat the gas as ideal
and monatomic. Make the approximation that the volume accessible to the gas is the volume V of the container,
independent of the much smaller volume occupied by the solid.
(a) Show that the total Helmholtz free energy of the system with the total number of atoms N = Ns +Ng is constant.
F = Fs +Fg = Nso +Ng [log (Ng/V nQ) 1] (12.1)
The entropy of the solid can be neglected, because it has no eect in the free energy. Fs = Us = Nso The free
energy of a gas is given in Kittel and Kroemer as Fg = Ng [log (n/nQ) 1] here n = Ng/V . The total free energy
is then
F = Nso +Ng [log (Ng/V nQ) 1]
42
(b) Find the minimum of the free energy with respect to Ng; show that in the equilibrium condition
Ng = nQV e
o/
(12.2)
At equilibrium
_
F
Ng
_
N,,V
= 0. Then

Ng
_
_
N Ng

o + Ng
_
log
_
Ng/V n
Q
_
1
__
= o +
_
log
_
Ng/V n
Q
_
1
_
+ = o + log
_
Ng/V n
Q
_
= 0
Finally
o + log (Ng/V nQ) = 0 Ng = nQV e
o/tau

(c) Find the equilibrium vapor pressure.


From part b and the ideal gas law
p =
Ng
V
= nQe
o/
p =
5/2
_
m
2
2
_
3/2
e
o/

43
44
Chapter 13
Problem Set 13
13.1 13.1: Superconduction and Heat Capacity
A certain metal in zero magnetic eld and at atmospheric pressure has a heat capacity Cn = in the normal state,
and a heat capacity Cs =
3
in the superconducting state. Here and are constants.
(a) Express these constants in terms of each other and the critical temperature c.
The heat capacity is given by Ci =
_

_
i
. Then
Cn =
n

=
n

n =
Cs =
s


3
=
s

s =
1
3

3
At the critical temperature n = s. Then
c =
1
3

3
c
=

3

2
c

(b) What is the dierence between the internal energy of the metal in the normal and superconducting states = 0?
Express the answer in terms of and c.
The internal energy dierence comes from Un =
_
c
0
Cnd = 1/2
2
c
and Us =
_
c
0
Csd = 1/4
4
c
. Then
U =
1
2

2
c

1
4

4
c
=
1
6

4
c

1
4

4
c
U =

12

4
c

13.2 13.2: Kittel 10.8: First order crystal transformation
Consider a crystal that can exist in either of two structures, denoted by and . We suppose that the structure
is the stable low temperature form and the structure is the stable high temperature form of the substance. If the
zero of the energy scale is taken as the state of separated atoms at innity, then the energy density U(0) at = 0
will be negative. The phase stable at = 0 will have the lower value of U(0); thus U(0) < U

(0). If the velocity of


sound v

in the phase is lower than v in the phase, corresponding to lower values of the elastic moduli for ,
then the thermal excitations in the phase will have larger amplitudes than in the phase. The larger the thermal
excitation, the large the entropy and the lower the free energy. Soft systems ten to be stable at high temperatures,
hard systems at low.
(a) Show from Chapter 4 that the free energy density contributed by the phonons in a solid at a temperature much
less than the Debye temperature is given by
2

4
/30v
3

3
, in the Debye approximation with v taken as the
velocity of all phonos.
The energy density of the phonons at is
U =
3
4
N
4
5 (kB)
3
=

2

4
10 (v)
3
45
Now
(F/) =
U

2
F/ =
_

2

2
10 (v)
3
d
F/ = U(0)

2

3
30 (v)
3
F = U(0)

2

4
30 (v)
3

(b) Show that at the transformation temperature

4
c
=
_
30
3
/
2
_
[U

(0) U(0)] /
_
v
3

v
3

_
(13.1)
There will be a nite real solution if v

< v. This example is simplied model of a class of actual phase


transformations in solids.
At the phase transformation the Gibbs free energy are equal. If there are no volume changes F(c) = F

(c). From
above
F(c) = F

(c) U(0)

2

4
c
30 (v)
3
= U

(0)

2

4
c
30 (v

)
3

4
c
_
1
v
3

1
v
3

_
=
30
3

2
(U

(0) U(0))
4
c
=
30
3

2
U

(0) U(0)
v
3

v
3

(c) The latent heat of transformation is dened as the thermal energy that must be supplied to carry the system
through the transformation. Show that the latent heat of this model is
L = 4 [U

(0) U(0)] (13.2)


The latent heat is the change in enthalpy. If the volume change is small, the change is just the change of internal
energy.
L = H

H = U

(c) U(c)
= U

(0) U(0)
_

2

4
c
10 (v)
3


2

4
c
10 (v

)
3
_
= U

(0) U(0) (U

(0) +U(0)) = 4 (U

(0) U(0))
13.3 13.3: Kittel 11.2: Mixing energy in
3
He
4
He and Pb Sn mixtures
The phase diagram of liquid
3
He
4
He mixtures in Figure 11.8 shows that the solubility of
3
He in
4
He remains nite
(about 6 percent) as 0. Similarly, the Pb Sn phase diagram of Figure 11.14 shows a nite residual solubility of
PB in solid Sn with decreasing . What do such nite residual solubilities imply about the form of the function u(x)?
The nite residual solubilities imply that at straight line can be drawn from x = 0 that is tangential to the minimum
of u(x), for Helium is x .94 and for Pb Sn x .97.
13.4 13.4: Kittel 11.4: Solidication range of a binary alloy
Consider the solidication of a binary alloy with the phase diagram of Figure 11.10. Show that, regardless of the
initial composition, the melt will always become fully depleted in component B by the time the last remnant of the
melt solidies. That is, the solidication will not be complete until the temperature has dropped to TA.
Starting as some concentration x, as it is cooled some will solidify, xS1. This amount is removed from x. From the
diagram, it will consist of B, so the remaining liquid, xL1, will have a lower concentration of B. Again, cooling the
sample more some more will solidify, xS2. Now the liquid, xL2, will have more of B removed, so xL2 < xL1. This can
be done continuously until all of B has been removed.
46
13.5 13.5: Kittel 11.5: Alloying of gold into silicon
(a) Suppose a 1000

A layer of Au is evaporated onto a Si crystal, and subsequently heated to 400
o
C. From the
Au Si phase diagram, Figure 11.11, estimate how deep the gold will penetrate into the silicon crystal. The
densities of Au and Si are 19.3 and 2.23 g cm
3
.
The total number of moles per unite area is n, and nAu, nSi, which are the number of moles per unite area of gold
and silicon, respectively. At a given temperature, the number of gold moles in a mixture is nAu = xAun. The same
can be said for silicon. The number of moles of gold per unit area is nAu = tAuAu/MAu. Then same can be side for
silicon. From gure 11.11, at 400
o
C, xSi = .32 xAu = .62. Then
nAu
xAu
=
nSi
xSi

tAuAu
MAuxAu
=
tSiSi
MSixSi
tSi =
tAuAuMSixSi
SiMSixAu
=
1000

A19.3g cm
3
28g mol
1
.32
2.33g cm
3
197g mol
1
.68
tSi = 554

A
(b) Redo the estimate for 800
o
C.
For 800
o
C, xSi = .45 then xAu = .55. Then
tSi =
tAuAuMSixSi
SiMSixAu
=
1000

A19.3g cm
3
28g mol
1
.45
2.33g cm
3
197g mol
1
.55
tSi = 963

A
47

You might also like